Find derivative of an integral with limits

Click For Summary
The discussion focuses on finding the derivative g'(x) of the function g(x), which is defined as the sum of two integrals with variable limits. The derivative is derived using Leibniz's rule, leading to the result g'(x) = f(x+δ) - f(x-δ). Concerns are raised about the behavior of the function at the lower limit of zero, particularly when the function f(x) could be undefined at that point. It is noted that the integral's behavior is contingent on the properties of f(x) and the conditions under which Leibniz's rule applies. The conversation emphasizes the importance of ensuring that the integrals involved are well-defined to avoid divergence issues.
Fire flame
Messages
8
Reaction score
0
I'm in analysis and I'm trying to understand the following.


Homework Statement



g(x) = integral from 0 to x+δ of f(x)dx + integral from x-δ to 0 of f(x)dx

g'(x) = f(x+δ) - f(x -δ)


So how do they get g'(x)?
 
Physics news on Phys.org
Fire flame said:
I'm in analysis and I'm trying to understand the following.


Homework Statement



g(x) = integral from 0 to x+δ of f(x)dx + integral from x-δ to 0 of f(x)dx

g'(x) = f(x+δ) - f(x -δ)


So how do they get g'(x)?

By Leibnitz rule for differentiation of an integral as a function of the upper limit.
By the way, it's better to use a dummy variable in the integrand:$$
\frac d {dx}\int_a^x f(t)\, dt = f(x)$$
 
So I understand why there isn't an f(a) since the derivative of a constant is zero, but like in my problem one of my limits is zero and since the function isn't given it could be anything, even something like f(x) = 1/x which at zero is undefined, but in my problem it just goes away to zero. Why? I hope you understand what I'm trying to say.
 
Fire flame said:
So I understand why there isn't an f(a) since the derivative of a constant is zero, but like in my problem one of my limits is zero and since the function isn't given it could be anything, even something like f(x) = 1/x which at zero is undefined, but in my problem it just goes away to zero. Why? I hope you understand what I'm trying to say.

There are hypotheses on Leibnitz's rule. You can't even talk about, for example, things like$$
F(x) = \int_0^x \frac 1 t\, dt$$because the integral is divergent.
 
Question: A clock's minute hand has length 4 and its hour hand has length 3. What is the distance between the tips at the moment when it is increasing most rapidly?(Putnam Exam Question) Answer: Making assumption that both the hands moves at constant angular velocities, the answer is ## \sqrt{7} .## But don't you think this assumption is somewhat doubtful and wrong?

Similar threads

  • · Replies 8 ·
Replies
8
Views
2K
Replies
5
Views
2K
Replies
2
Views
2K
  • · Replies 2 ·
Replies
2
Views
2K
  • · Replies 16 ·
Replies
16
Views
2K
Replies
9
Views
1K
  • · Replies 10 ·
Replies
10
Views
2K
  • · Replies 14 ·
Replies
14
Views
2K
Replies
2
Views
1K
Replies
30
Views
3K